Discrete Structures Subject Wise UGC NET Question Analysis Part-1

Discrete Math PYQ Part-2


Q.1➡ | UGC NET DEC 2023
If N²=NxN, N is set of natural numbers and R is relation on N2, s.t. RC N² × N²i.e. <x,y> R<u,v> xv=yu, then which of the followings are TRUE ?
(A) Reflexive
(B) Symmetric
(C) Transitive
(D) Assymmetric
Choose the correct answer from the options given below :
i ➥ (A) and (B) Only
ii ➥ (B) and (C) Only
iii ➥ (A), (C) and (D) Only
iv ➥ (A), (B) and (C) Only
Best Explanation:
Answer: (IV)
Explanation: Upload Soon
More DiscussionExplanation On YouTubeLearn Topic WiseHelp-Line


Q.2➡ | UGC NET DEC 2023
If universe of disclosure are all real numbers, then which of the following are true?
(A) Ǝx ∀y (x+y=y)
(B) ∀x ∀y(((x≥0)∧(y<0))→(x-y>0))
(C) Ǝx Ǝy(((x≤0)∧(y≤0))∧(x-y>0))
(D) ∀x ∀y((x≠0)∧(y≠0)↔(xy≠0))
Choose the correct answer from the options given below :
i ➥ (A) and (B) Only
ii ➥ (A), (C) and (D) Only
iii ➥ (A), (B) and (D) Only
iv ➥ (A), (B), (C) and (D) Only
Best Explanation:
Answer: (IV)
Explanation: Upload Soon
More DiscussionExplanation On YouTubeLearn Topic WiseHelp-Line

Q.3➡ | UGC NET DEC 2023
If the universe of disclosure is set of integers, then which of the followings are TRUE?
(A) n m(n2 < m)
(B) n m(n < m2)
(C) n m(nm = m)
(D) n m(n2 + m2 = 6)
(E) 3n m(n + m = 4 ∧ n – m = 1)
Choose the correct answer from the options given below :
i ➥ (A), (B) and (C) Only
ii ➥ (B) and (C) Only
iii ➥ (C), (D) and (E) Only
iv ➥ (C) and (E) Only
Best Explanation:
Answer: (II)
Explanation: Upload Soon
More DiscussionExplanation On YouTubeLearn Topic WiseHelp-Line


Q.4➡ | UGC NET DEC 2023
Which of the following statement are truth statements if universe of disclosure is set of integers:
(A) ∀n(n2 >= 0)
(B) ∃ n(n 2 = 2)
(C) ∀ n(n 2 >= n)
(D) ∃ n(n2 < 0)
Choose the correct answer from the options given below :
i ➥ (A) and (B) Only
ii ➥ (B) and (C) Only
iii ➥ (C) and (D) Only
iv ➥ (A) and (C) Only
Best Explanation:
Answer: (IV)
Explanation: Upload Soon
More DiscussionExplanation On YouTubeLearn Topic WiseHelp-Line


Q.5➡ | UGC NET DEC 2023
Which of the following are tautology?
(A) (P (P∧ Q))→(P→Q)
(B) ((PQ)→Q)→(PVQ)
(C) ((P V¬P)→Q)→((PV ¬P)→R)
(D) (Q→(P∧ ¬P))→(R→(P∧ ¬P))
Choose the correct answer from the options given below :
i ➥ (A) Only
ii ➥ (B) Only
iii ➥ (A) and(B) Only
iv ➥ (C) and (D) Only
Best Explanation:
Answer: (III)
Explanation: Upload Soon
More DiscussionExplanation On YouTubeLearn Topic WiseHelp-Line


Q.6➡ | UGC NET DEC 2023
The statement P(x): “x=x2“. If the universe of disclosure consists of integers, what are the following have truth values :
(A) P(0)
(B) P(1)
(C) P(2)
(D) Ǝx P(x)
(E) ∀x P(x)
Choose the correct answer from the options given below :
i ➥ (A), (B) and (E) Only
ii ➥ (A), (B) and (C) Only
iii ➥ (A), (B) and (D) Only
iv ➥ (B), (C) and (D) Only
Best Explanation:
Answer: (IV)
Explanation: Upload Soon
More DiscussionExplanation On YouTubeLearn Topic WiseHelp-Line

Q.7➡ | UGC NET DEC 2023
What is the probability that a positive integer selected at random from the set of positive integer not exceeding 100 is divisible by either 2 or 5?
i ➥ 10/5
ii ➥ 3/5
iii ➥ 2/5
iv ➥ 1/5
Best Explanation:
Answer: (II)
Explanation: Upload Soon
More DiscussionExplanation On YouTubeLearn Topic WiseHelp-Line


Q.8➡ | UGC NET DEC 2023
Given below are two statements:
Statement (I): If H is non empty finite subset of a group G and ab∉H ∀ a, b∉H, then H is also a group
Statement (II): There is no homomorphism exist from (Z, +) to (Q, +); where Z is set of integers and Q is set of rational number.
In the light of the above statements, choose the most appropriate answer from the options given below:
i ➥ Both Statement I and Statement II are correct
ii ➥ Both Statement I and Statement II are incorrect
iii ➥ Statement I is correct but Statement II is incorrect
iv ➥ Statement I is incorrect but Statement II is correct
Best Explanation:
Answer: (III)
Explanation: Upload Soon
More DiscussionExplanation On YouTubeLearn Topic WiseHelp-Line


Q.9➡ | UGC NET JUNE 2023
Let R = {x | x ∈ N, x is multiple of 3 and x ≤ 100} and S = {x, x ∈ N, x is a multiple of 5 and x < 100}. What is the number of elements in (R∩S) × (S∩R)?
i ➥ 36
ii ➥ 33
iii ➥ 20
iv ➥ 6
Best Explanation:
Answer: I
Explanation:

Set R contains elements: (3, 6, 9, 12, 15, 18, 21, 24,….93, 96, 99)
Set S contains elements: (5, 10, 15, 20, 25, 30,…85, 90, 95)
R cap S = {15, 30, 45, 60, 75, 90}
S cap R = {15, 30, 45, 60, 75, 90}
Since the cardinality (number of elements in the sets) of both sets are 6.
So (R∩S) x (S∩R) = 6*6 = 36
Option (A) is correct.
More DiscussionExplanation On YouTubeLearn Topic WiseHelp-Line

Q.10➡ | UGC NET JUNE 2023
Match List I with List II

List I
A. A Δ B
B. A−(B∪C)
C. A−(B∩C)
D. A∩(B−C)

List II
I. (A−B)∪(A−C)
II. (A−B)∩(A−C)
III. (A−B)∪(B−A)
IV. (A∩B)−(A∩C)

Choose the correct answer from the options given below: Choose the correct answer from the options given below:
i ➥ A-III B-II C-I D-IV
ii ➥ A-II B-III C-IV D-I
iii ➥ A-IV B-III C-I D-II
iv ➥ A-IV B-I C-III D-II
Best Explanation:
Answer: I
Explanation: Upload Soon
More DiscussionExplanation On YouTubeLearn Topic WiseHelp-Line

Q.11➡ | UGC NET JUNE 2023
Let N denote the set of all natural numbers and R be the relation on N*N defined by (a,b)R(c,d), if ad(b+c)=bc(a+d). Then R is
i ➥ Symmetric only
ii ➥ Reflexive only
iii ➥ Transitive only
iv ➥ An equivalence relation
Best Explanation:
Answer: IV
Explanation: Upload Soon
More DiscussionExplanation On YouTubeLearn Topic WiseHelp-Line
Q.12➡ | UGC NET JUNE 2023
Let (Z,+) denote the group of all integers under addition. Then the number of all automorphisms of (Z,+) is
i ➥ 1
ii ➥ 2
iii ➥ 3
iv ➥ 4
Best Explanation:
Answer: II
Explanation:

In the case of the group of all integers under addition, (Z, +), the number of automorphisms is 2.
1.The identity function: The identity function is an automorphism that maps each integer to itself. It preserves the group’s operation and structure.
2. The negation function: The negation function is also an automorphism. It maps each integer to its additive inverse, effectively reversing the sign. This function also preserves the group’s operation and structure.
These two automorphisms are the only possible automorphisms for the group (Z, +). So, the number of automorphisms of (Z, +) is 2.
More DiscussionExplanation On YouTubeLearn Topic WiseHelp-Line
Q.13➡ | UGC NET JUNE 2023
If A={4n +2|n is a natural number } and B = {3n | n is a natural number }. Which of the following is correct for A ∩ B ?
i ➥ { 12n ^ 2 +6n|n is a natural number }
ii ➥ {24n 12 n is a natural number }
iii ➥ { 60n +30|n is a natural number }
iv ➥ { 12n -6|n is a natural number }
Best Explanation:
Answer: IV
Explanation: Upload Soon
More DiscussionExplanation On YouTubeLearn Topic WiseHelp-Line

Q.14➡ | UGC NET JUNE 2023
i ➥ Not a homomorphism
ii ➥ A one-one homomorphism, which is not onto
iii ➥ An onto homomorphism, which is not one to one
iv ➥ An homomorphism
Best Explanation:
Answer: III
Explanation: Upload Soon
More DiscussionExplanation On YouTubeLearn Topic WiseHelp-Line
Q.15➡ | UGC NET JUNE 2023
Consider the following statements.
A. The identity is unique in any monoid.
B. A monoid is a group if there exists inverse of each element of monoid.
C. Semi group has closure, associative and identity properties.
D. Quasi group has closure property.


Choose the correct answer from the options given below :
i ➥ A,B and D only
ii ➥ B,C and D only
iii ➥ A,B and C only
iv ➥ A,C and D only
Best Explanation:
Answer: I
Explanation: Upload Soon
More DiscussionExplanation On YouTubeLearn Topic WiseHelp-Line
Q.16➡ | NET December 2022
A relation ‘R’ is defined on ordered pairs of integers as : (x, y) R (u, v) if x<u and y>v. Then R is
i ➥ Neither a partial order nor an equivalence relation
ii ➥ A partial order but not a total order
iii ➥ A total order
iv ➥ An equivalence relation
Best Explanation:
Answer: (i)
Explanation: Upload Soon
More DiscussionExplanation On YouTubeLearn Topic WiseHelp-Line

Q.17➡ | NET December 2022
Suppose you are married and you and your partner attend a party with three other married couples. Several handshakes took place. No one shook hands with himself ( or herself ) or with their partner, and no one shook hands with the same person more than once. After all hand shaking was completed , suppose you asked each person, including your partner, how many hands they had shaken. Each person gave a different answer. How many hands did your spouse shake?
i ➥ 1
ii ➥ 2
iii ➥ 3
iv ➥ 4
Best Explanation:
Answer: (iii)
Explanation: Upload Soon
More DiscussionExplanation On YouTubeLearn Topic WiseHelp-Line

Q.18➡ | NET December 2022
Consider the following statements:
P : There exists no simple, undirected and connected graph with 80 vertices and 77 edges
Q : All vertices of Euler graph are of even degree
R : Every simple, undirected, connected and acyclic graph with 50 vertices has at least two vertices of degree one
S : There exists a bipartite graph with more than ten vertices which is 2 – colorable.
What is the number of correct statements among the above statements.
i ➥ 1
ii ➥ 2
iii ➥ 3
iv ➥ 4
Best Explanation:
Answer: (iv)
Explanation: Upload Soon
More DiscussionExplanation On YouTubeLearn Topic WiseHelp-Line

Q.19➡ | NET December 2022
Q42➡ | Discrete Mathematics
consider the following statements.
A : There exists a Boolean algebra with 5 elements
B : Every element of Boolean algebra has unique complement
C : It a Lattice ‘L’ is a Boolean algebra then ‘L’ is not relatively complemented
D : The direct product of two Boolean algebra is also a Boolean algebra
Choose the correct answer about the four statements given above.
i ➥ Only A and D are correct
ii ➥ Only B and D are correct
iii ➥ All statement are NOT Correct
iv ➥ All statement are correct
Best Explanation:
Answer: (ii)
Explanation: Upload Soon
More DiscussionExplanation On YouTubeLearn Topic WiseHelp-Line

Q.20➡ | NET December 2022
Given a vector with Cartesian components (x,y) , if scaling is done with Matrix Row1[ 0.5 0 ], Row2[0 1.5] which of the following are true:
(A) Decrease the vertical by three halves
(B) Increase the vertical by three halves
(C) Double the horizontal
(D) Halves the horizontal
Choose the correct from the options given below:
i ➥ A and C only
ii ➥ A and D only
iii ➥ B and C only
iv ➥ B and D only
Best Explanation:
Answer: (iv)
Explanation: Upload Soon
More DiscussionExplanation On YouTubeLearn Topic WiseHelp-Line

Q.21➡ | NET December 2022
Match List (I) with List (II)
A.	 Planner graph	I.	Probabilistic Model
B.	Bipartite graph	II.	Deterministic Model
C. 	PERT	III.	4-Colorable
D. 	CPM	IV.	2-colorable
Choose the correct answer from the options given below:
i ➥ A-(IV), B-(III), C-(I), D-(II)
ii ➥ A-(III), B-(IV), C-(II), D-(I)
iii ➥ A-(II), B-(IV), C-(I), D-(III)
iv ➥ A-(III), B-(IV), C-(I), D-(II)
Best Explanation:
Answer: (iv)
Explanation: Upload Soon
More DiscussionExplanation On YouTubeLearn Topic WiseHelp-Line

Q.22➡ | NET December 2022
Statement 1:
Given a graph G=(VE) in which each vertex v € V has an associated positive weight w(v), we can use linear programming to find the lower bound on the weight of the minimum-weight vertex cover.
Statement 2:
The lower bound can be found by maximizing the following
∑_vv^n▒〖w(v)x(v)〗 .
Subject to
x(u) + x(v) 1 for each (u,v) V
            x (v)1 for each v  V
           x (v)  0 for each v  V
In the light of the above statements, choose the most appropriate answer from the options given below:
i ➥ Both statement (I) and Statement (II) are correct
ii ➥ Both statement (I) and Statement (II) are incorrect
iii ➥ Statement (I) is correct but Statement (II) is incorrect
iv ➥ Statement (I) is incorrect and Statement (II) is correct
Best Explanation:
Answer: (iii)
Explanation: Upload Soon
More DiscussionExplanation On YouTubeLearn Topic WiseHelp-Line

Q.23➡ | NET June 2022
Let ({a,b}, ) be a semigroup , where a*a = b.
(A) a*a = b*a
(B) b*b = b
Choose the most appropriate answer from the options given below:
i ➥ A) only true
ii ➥ (B) only true
iii ➥ Both (A) and (B) true
iv ➥ Neither (A) not (B) true
Best Explanation:
Answer: (iii)
Explanation: Upload Soon
More DiscussionExplanation On YouTubeLearn Topic WiseHelp-Line

Q.24➡ | NET June 2022
Consider the primal problem:
Maximize z = 5x1 + 12x2 + 4x3
Subject to x1 + 2x2 + x3 = 10
2x1 – x2 + 3x3 = 8
x1, x2, x3 ≥ 0
It’s dual problem is
Minimize w = 10y1 + 8y2
Subject to y1 + 2y2 ≥ 5
2y1 – y2 ≥ 12
y1 + 3y2 ≥ 4
Which of the following is correct?
i ➥ y1≥ 0 , y2 unrestricted
ii ➥ y1≥ 0 , y2≥ 0
iii ➥ y1 is unrestricted, y2 ≥ 0
iv ➥ y1 is unrestricted, y2 is restricted
Best Explanation:
Answer: (i)
Explanation: Upload Soon
More DiscussionExplanation On YouTubeLearn Topic WiseHelp-Line

Q.25➡ | NET June 2022
There are three boxes. First box has 2 white, 3 black and 4 red balls. Second box has 3 white, 2 Black and 2 red balls. Third box has 4 white, 1 black and 3 red balls. A box is chosen at random and 2 balls are drawn out of which 1 is white , and 1 is red. What is the probability that the balls came from first box?
i ➥ 0.237
ii ➥ 0.723
iii ➥ 0.18
iv ➥ 0.452
Best Explanation:
Answer: (i)
Explanation: Upload Soon
More DiscussionExplanation On YouTubeLearn Topic WiseHelp-Line

Q.26➡ | NET June 2022
Let ∈ = 0.0005 and let Re be the relation { (x, y) ∈ R2 : |x-y| <∈} , Re could be interpreted as the relation approximately equal Re is
(A) Reflexive
(B) Symmetric
(C) Transitive
Choose the correct answer from the option given below :
i ➥ (A) and (B) only True
ii ➥ (B) and (C) only True
iii ➥ (A) and (C) only True
iv ➥ (A), (B) and (C) True
Best Explanation:
Answer: (i)
Explanation: Upload Soon
More DiscussionExplanation On YouTubeLearn Topic WiseHelp-Line

Q.27➡ | NET June 2022
Consider the following Boolean algebra 
X : a  ( b  (a  c )) = ( a  b ) ( a  c )
Y : a  ( b  ( a  c )) =  ( a  b )  ( a  c )
a(bc) = (ab)c  is satisfied if
i ➥ X is true
ii ➥ Y is true
iii ➥ Both X and Y are True
iv ➥ It does not depend on X and Y
Best Explanation:
Answer: (iii)
Explanation: Upload Soon
More DiscussionExplanation On YouTubeLearn Topic WiseHelp-Line

Q.28➡ | NET June 2021
Let (X,* ) be a semi-group. Furthermore, for every a and b in X, if a ≠ b, then a*b ≠ b*a. Based on the defined semi-group,
choose the correct equalities from the options given below:
A. For every a in X, a*a = a
B. For every a, b in X, a*b *a= a
C. For every a, b, c in X, a*b *c= a*c
i ➥ A and B only
ii ➥ A and C only
iii ➥ A, B and C
iv ➥ B and C only
Answer: III

Explanation:
Given,
(X,* ) be a semi-group.
for every a and b in X, if a ≠ b, then a*b ≠ b*a.

Concept,
Semi Group(X,* ) have 2 properties, which are:
1) Closure Property
if a and b in X, then a*b∈X

2) Associative Property
if a,b and c in X, then
(a*b)*c = a*(b*c)

Let’s solve,
A) For every a in X, a*a = a

Lets assume a*a ≠ a

Lets suppose a*a = b, where a ≠ b
⟹ a*(a*a) = a*b
⟹ (a*a)*a = a*b { by associative property, a*(b*c) = (a*b)*c}
⟹ b*a = a*b { we suppose a*a = b}

Since it is given, for every a and b in X, if a ≠ b, then a*b ≠ b*a, but here a*b = b*a, which proves that our initial assumption is wrong.
Hence, For every a in X, a*a = a (Correct)

B) For every a, b in X, a*b *a= a

Lets assume a*b*a ≠ a

Lets suppose a*b*a = b, where a ≠ b

it is given, a*b ≠ b*a, and we already suppose a*b*a = b. Now put b = a*b*a in a*b ≠ b*a
⟹ a*(a*b*a) ≠ (a*b*a)*a
⟹ (a*a)*b*a ≠ a*b*(a*a) { by associative property, a*(b*c) = (a*b)*c}
⟹ a*b*a ≠ a*b*a { it is given in A, a*a = a and we already proved that this is correct}

Here, a*b*a ≠ a*b*a but it is actually equal, which proves that our initial assumption is wrong.
Hence, For every a, b in X, a*b *a= a (Correct)

C) For every a, b, c in X, a*b *c= a*c

Lets assume a*b*c ≠ a*c

Lets suppose a*b*c = a, and a*c=b where a ≠ b

it is given, a*b ≠ b*a, and we already suppose a*b*c = a and a*c=b . Now put a= a*b*c and b = a*c in a*b ≠ b*a
⟹ (a*b*c)*(a*c) ≠ (a*c)*(a*b*c)
⟹ a*b*(c*a*c) ≠ (a*c*a)*b*c { by associative property, a*(b*c) = (a*b)*c}
⟹ a*b*c ≠ a*b*c { it is given in B, a*b *a= a and we already proved that this is correct. Like statement B, we can write c*a*c=c and a*c*a=a }

Here, a*b*c ≠ a*b*c but it is actually equal, which proves that our initial assumption is wrong.
Hence, For every a, b in X, a*b *a= a (Correct)

Therefore, Option(III) is correct.
[Easy Solution]Explanation On YouTubeGroupHelp-Line

Q.29➡ | NET June 2021
How many ways are there to assign 5 different jobs to 4 different employees if every employee is assigned at least 1 job?
i ➥ 1024
ii ➥ 20
iii ➥ 240
iv ➥ 625
Answer: III

Explanation:
Given,
5 different jobs and 4 different employees
every employee is assigned at least 1 job


Let’s solve,
Lets take 4 employees as A,B,C,D

  How many ways are there to assign 5 different jobs to 4 different employees if every employee is assigned at least 1 job?

Total ways = 60 + 60 + 60 + 60 = 240 ways

Therefore, Option(III) is correct.
[Easy Solution]Explanation On YouTubePermutation and CombinationHelp-Line

Q.30➡ | NET June 2021
Consider the sentence below.
There is a country that borders both India and Pakistan.
Which of the following logical expressions express the above sentence correctly when the predicate Country(x)
represents that x is a country and Borders(x, y) represents that the countries x and y share the border?
i ➥ [∃c Country(c)] ⇒ [Border (c, India) ∧ Border (c,Pakistan)]
ii ➥ ∃c Border (Country(c), India ∧ Pakistan)
iii ➥ ∃c Country(c) ∧ Border (c, India) ∧ Border (c,Pakistan)
iv ➥ ∃c Country(c) ⇒ [Border (c, India) ∧ Border (c,Pakistan)]

Show Answer With Best Explanation

Answer: III
Explanation: Upload Soon

More DiscussionExplanation On YouTubePropositional logicHelp-Line

Q.31➡ | NET June 2021
Which of the following Graphs is (are) planar?

choose the correct answer from the options given below:
i ➥ A and B only
ii ➥ A only
iii ➥ B and C only
iv ➥ B only
Answer: II
More DiscussionExplanation On YouTubeGraph Theory Help-Line

Q.32➡ | NET June 2021
which of the following are logically equivalent ?
A. ¬p→(q→r)and q→(p∨r)
B. (p→q)→r and p→(q→r)
C. (p→q)→(r→s) and (p→r)→(q→s)

Choose the correct answer from the options below:
i ➥ A and B only
ii ➥ A and C only
iii ➥ A only
iv ➥ B and C only

Show Answer With Best Explanation

Answer: III
Explanation: Upload Soon

More DiscussionExplanation On YouTubeLearn Topic WiseHelp-Line

Q.33➡ | NET June 2021
For which value of n is Wheel graph Wn regular?
i ➥ 2
ii ➥ 3
iii ➥ 4
iv ➥ 5

Show Answer With Best Explanation

Answer: II
Explanation: Upload Soon

More DiscussionExplanation On YouTubeLearn Topic WiseHelp-Line

Q.34➡ | NET June 2021
Let us assume a person climbing the stairs can take one stair or two stairs at a time.
How many ways can this person climb a flight of eight stairs?
i ➥ 21
ii ➥ 24
iii ➥ 31
iv ➥ 34

Show Answer With Best Explanation

Answer: IV
Explanation: Upload Soon

More DiscussionExplanation On YouTubeLearn Topic WiseHelp-Line

Q.35➡ | NET June 2021
Next five questions are based on the following passage.
Consider a domain consisting of three Boolean variables Toothache, Cavity, and Catch. The full joint distribution is a 2×2×2 table as shown in the figure below.

The probability of a toothache, given evidence of a cavity, P(toothache | cavity) is ____.
i ➥ 0.216
ii ➥ 0.280
iii ➥ 0.400
iv ➥ 0.600

Show Answer With Best Explanation

Answer: IV
Explanation:
Formula,
Conditional Probability
The probability of a toothache, given evidence of a cavity, P(toothache | cavity) is ____.
Calculation,
The probability of a toothache, given evidence of a cavity,
 The probability of a toothache, given evidence of a cavity, P(toothache | cavity) is ____.
The probability of a toothache, given evidence of a cavity, P(toothache | cavity) is ____.
P(toothache Ո cavity ) = 0.108 + 0.012 = 0.12

The probability of a toothache, given evidence of a cavity, P(toothache | cavity) is ____.
P(cavity ) = 0.108 + 0.012 + 0.072 + 0.008 = 0.2
 The probability of a toothache, given evidence of a cavity, P(toothache | cavity) is ____.
The probability of a toothache, given evidence of a cavity, P(toothache | cavity) is ____.
So,Option(IV) is correct.

More Discussion Explanation On YouTubeProbability Help-Line

Q.36➡ | NET June 2021
Next five questions are based on the following passage.
Consider a domain consisting of three Boolean variables Toothache, Cavity, and Catch. The full joint distribution is a 2×2×2 table as shown in the figure below.

P(cavity U toothache) is________________.
i ➥ 0.120
ii ➥ 0.200
iii ➥ 0.280
iv ➥ 0.600

Show Answer With Best Explanation

Answer: III
Explanation:
Formula,
P(A U B) = P(A) + P(B) – P(A Ո B)
Calculation,
P(cavity U toothache) = P(cavity) + P(toothache) – P(cavity Ո toothache)
The probability of a toothache, given evidence of a cavity, P(toothache | cavity) is ____.
P(cavity ) = 0.108 + 0.012 + 0.072 + 0.008 = 0.2

P(cavity U toothache) is________________.
P(toothache) = 0.108 + 0.012 + 0.016 + 0.064 = 0.2


P(cavity Ո toothache) = 0.108 + 0.012 = 0.12

P(cavity U toothache) = P(cavity) + P(toothache) – P(cavity Ո toothache)
P(cavity U toothache) = 0.2 + 0.2 – 0.12 = 0.28
So, Option(III) is correct.

More Discussion Explanation On YouTubeProbability Help-Line

Q.37➡ | NET June 2021
Next five questions are based on the following passage.
Consider a domain consisting of three Boolean variables Toothache, Cavity, and Catch. The full joint distribution is a 2×2×2 table as shown in the figure below.

The probability for Cavity, given that either Toothache or Catch is true, P(Cavity | toothache U catch) is _______.
i ➥ 0.4615
ii ➥ 0.5384
iii ➥ 0.6000
iv ➥ 0.8000

Show Answer With Best Explanation

Answer: I
Explanation:
Formula,
The probability for Cavity, given that either Toothache or Catch is true, P(Cavity | toothache U catch) is _______.
Calculation,
 The probability for Cavity, given that either Toothache or Catch is true, P(Cavity | toothache U catch) is _______.
The probability for Cavity, given that either Toothache or Catch is true, P(Cavity | toothache U catch) is _______.
P(cavity Ո (toothache U catch)) = 0.108 + 0.012 + 0.072 = 0.192

 The probability for Cavity, given that either Toothache or Catch is true, P(Cavity | toothache U catch) is _______.
P(toothache U cavity) = 0.108 + 0.012 + 0.072 + 0.016 + 0.064 + 0.144 = 0.416

 The probability for Cavity, given that either Toothache or Catch is true, P(Cavity | toothache U catch) is _______.
 The probability for Cavity, given that either Toothache or Catch is true, P(Cavity | toothache U catch) is _______.= 0.4615
So, Option(I) is correct.

More Discussion Explanation On YouTubeProbability Help-Line

` Q.38➡ | NET June 2021
Next five questions are based on the following passage.
Consider a domain consisting of three Boolean variables Toothache, Cavity, and Catch. The full joint distribution is a 2×2×2 table as shown in the figure below.

The marginal probability of cavity P(cavity) is _____.
i ➥ 0.080
ii ➥ 0.120
iii ➥ 0.200
iv ➥ 0.216

Show Answer With Best Explanation

Answer: III
Explanation:
Calculation,
The probability of a toothache, given evidence of a cavity, P(toothache | cavity) is ____.
P(cavity ) = 0.108 + 0.012 + 0.072 + 0.008 = 0.2

So, Option(III) is correct.

More DiscussionExplanation On YouTubeProbability Help-Line

Q.39➡ | NET June 2021
Next five questions are based on the following passage.
Consider a domain consisting of three Boolean variables Toothache, Cavity, and Catch. The full joint distribution is a 2×2×2 table as shown in the figure below.

The probability of a cavity, given evidence of a toothache, P(cavity | toothache) is ____.
i ➥ 0.216
ii ➥ 0.280
iii ➥ 0.400
iv ➥ 0.600

Show Answer With Best Explanation

Answer: IV
Explanation:
Formula,
Conditional Probability
The probability of a toothache, given evidence of a cavity, P(toothache | cavity) is ____.
Calculation,
The probability of a cavity, given evidence of a toothache,
 The probability of a cavity, given evidence of a toothache, P(cavity | toothache) is ____.
 The probability of a cavity, given evidence of a toothache, P(cavity | toothache) is ____.
P(cavity Ո toothache ) = 0.108 + 0.012 = 0.12

P(cavity U toothache) is________________.
P(toothache) = 0.108 + 0.012 + 0.016 + 0.064 = 0.2
 The probability of a cavity, given evidence of a toothache, P(cavity | toothache) is ____.
 The probability of a cavity, given evidence of a toothache, P(cavity | toothache) is ____.= 0.6
So,Option(IV) is correct.

More Discussion Explanation On YouTubeProbability Help-Line

Q.40➡ | NET November 2020
Given below are two statements:
Statement I: 5 divides n5-n whenever n is a nonnegative integer.
Statement II: 6 divides n3-n whenever n is a nonnegative integer.

In the light of the above statements, choose the correct answer from the options given below
i➥ Both Statement I and Statement II are correct
ii ➥ Both Statement I and Statement II are incorrect
iii ➥ Statement I is correct but Statement II is incorrect
iv ➥ Statement I is incorrect but Statement II is correct

Show Answer With Best Explanation

Answer: I
Explanation:
Statement I: n5 – n
Example : Take n = 2 , 25- 2 = 32 – 2 = 30 , which is divided by 5.
Take n = 3 , 35 – 3 = 243 – 3 = 240 , which is divided by 5.
Statement I is true.
Statement II: n3 – n
Example : Take n = 2 , 23- 2 = 8 – 2 = 6 , which is divided by 6.
Take n = 3 , 33 – 3 = 27 – 3 = 24 , which is divided by 6.
Statement II is true.

More Discussion Explanation On YouTubeLearn Topic WiseHelp-Line

Q.41➡ | NET November 2020
Find the lexicographic ordering of the bit strings given below based on the ordering 0 < 1.
A) 001
B) 010
C) 011
D) 0001
E) 0101
Choose the correct answer from the options given below:
i➥ 001<010<011<0001<0101
ii ➥ 0001<001<010<0101<011
iii ➥ 0001<0101<001<010<011
iv ➥ 001<010<0001<0101<011

Show Answer With Best Explanation

Answer: II
Explanation:
Lexicographically means sorting in natural order, dictionary order.
The lexicographic order of the given bit strings will be:
0001<001<010<0101<011
Take 0001 & 001 for comparing. Compare each bit of 1st string with the corresponding bit of 2nd string.

It is clear that 0001< 001.

More DiscussionExplanation On YouTubeLearn Topic WiseHelp-Line

Q.42➡ | NET November 2020
Consider the following statements:
A) Any tree is 2-colorable
B) A graph G has no cycles of even length if it is bipartite
C) A graph G is 2-colorable if is bipartite
D) A graph G can be colored with d+1 colors if d is the maximum degree of any vertex in the graph G
E) A graph G can be colored with O(log|v|) colors if it has O(|v|) edges.

Choose the correct answer from the options given below:
i➥ (C) and (E) are incorrect
ii ➥ (B) and (C) are incorrect
iii ➥ (B) and (E) are incorrect
iv ➥ (A) and (D) are incorrect

Show Answer With Best Explanation

Answer: III
Explanation: Upload Soon

More DiscussionExplanation On YouTubeLearn Topic WiseHelp-Line

Q43➡ | NET November 2020
Let) G be a directed graph whose vertex set is the set of numbers from 1 to 100. There is an edge from a vertex i to a vertex j if and only if either j=i+1 or j=3i. The minimum number of edges in a path in G from vertex 1 to vertex 100 is ___________.
i➥ 23
ii ➥ 99
iii ➥ 4
iv ➥ 7

Show Answer With Best Explanation

Answer: IV
Explanation:
Edge set consists of edge from i to j , if and only if
either j=i+1
or j=3i
since, we to find the minimum number of edge from vertex 1 to vertex 100.so, we have to think about how we can reach an edge 100 from an edge 1 with minimum path.
1 ->3 ->9 -> 10 -> 11 -> 33 -> 99 -> 100.
We need minimum 7 edges.

More DiscussionExplanation On YouTubeLearn Topic WiseHelp-Line

Q.44➡ | NET November 2020
How many ways are there to pack six copies of the same book into four identical boxes, where a box can contain as many as six books?
i➥ 4
ii ➥ 6
iii ➥ 7
iv ➥ 9

Show Answer With Best Explanation

Answer: IV
Explanation: Upload Soon

More DiscussionExplanation On YouTubeLearn Topic WiseHelp-Line

Q.45➡ | NET November 2020
The number of positive integers not exceeding 100 that are either odd or the square of an integer is _.
i➥ 63
ii ➥ 59
iii ➥ 55
iv ➥ 50

Show Answer With Best Explanation

Answer: III
Explanation: Upload Soon

More DiscussionExplanation On YouTubeLearn Topic WiseHelp-Line

Q.46➡ | NET December 2019
Consider the following statements:
S1: If a group (G,) is of order n, and a ∈ G is such that am = e for some integer m ≤ n, then m must divide n.
S2: If a group (G,) is of even order , then there must be an element and a ∈ G is such that a ≠ e and a * a = e.

Which of the statements is (are) correct?
i➥ Only S1
ii ➥ Only S2
iii ➥ Both S1 and S2
iv ➥ Neither S1 nor S2

Show Answer With Best Explanation

Answer: III
Explanation: Upload Soon

More DiscussionExplanation On YouTubeLearn Topic WiseHelp-Line

Q.47➡ | NET December 2019
Let a2e mod n = (ae)2 mod n and a2c+1 mod n = a (ac)2 mod n. For a = 7, b = 17 and n = 561, what is the value of ab(mod n)?
i➥ 160
ii ➥ 166
iii ➥ 157
iv ➥ 67

Show Answer With Best Explanation

Answer: I
Explanation: Upload Soon

More DiscussionExplanation On YouTubeLearn Topic WiseHelp-Line

Q.48➡ | NET December 2019
Let P be the set of all people. Let R be a binary relation on P such that (a, b) is in R if a is a brother of b. Is R symmetric, transitive, an equivalence relation, a partial order relation?
i➥ NO.NO.NO.NO
ii ➥ NO.NO.YES.NO
iii ➥ NO.YES.NO.NO
iv ➥ NO.YES.YES.NO

Show Answer With Best Explanation

Answer: III
Explanation: Upload Soon

More DiscussionExplanation On YouTubeLearn Topic WiseHelp-Line

Q.49➡ | NET December 2019
What are the greatest lower bound (GLB) and the least upper bound (LUB) of the sets A = {3, 9, 12} and B = {1, 2, 4, 5, 10} if they exist in poset (z*,/)?
i➥ A (GLB – 3, LUB – 36); B (GLB – 1, LUB – 20)
ii ➥ A(GLB – 3, LUB – 12); B (GLB -1, LUB – 10)
iii ➥ A (GLB -1, LUB – 36); B (GLB – 2, LUB – 20)
iv ➥ A (GLB – 1, LUB – 12); B (GLB – 2, LUB – 10)

Show Answer With Best Explanation

Answer: I
Explanation: Upload Soon

More DiscussionExplanation On YouTubeLearn Topic WiseHelp-Line

Q.50➡ | NET December 2019
A clique in an undirected graph G = (V, E) is a subset V’ ⊆ V of vertices, such that
i➥ If (u, v) ∈ E then u ∈ V’ and v ∈ V’
ii ➥ If (u, v) ∈ E then u ∈ V’ or v ∈ V’
iii ➥ Each pair of vertices in V’ is connected by an edge
iv ➥ All pairs of vertices in V’ are not connected by an edge

Show Answer With Best Explanation

Answer: III
Explanation: Upload Soon

More DiscussionExplanation On YouTubeLearn Topic WiseHelp-Line

Q.51➡ | NET December 2019
How many reflexive relations are there on a set with 4 elements?
i➥ 24
ii ➥ 212
iii ➥ 42
iv ➥ 2

Show Answer With Best Explanation

Answer: II
Explanation: Upload Soon

More DiscussionExplanation On YouTubeLearn Topic WiseHelp-Line

Q.52➡ | NET June 2019
How many are there to place 8 indistinguishable balls into four distinguishable bins?
i➥ 70
ii ➥ 165
iii ➥ 8C4
iv ➥ 8P4 T

Show Answer With Best Explanation

Answer: II
Explanation: Upload Soon

More DiscussionExplanation On YouTubeLearn Topic WiseHelp-Line

Q.53➡ | NET June 2019
Suppose that a connected planar graph has six vertices, each of degrees four. Into how many regions is the plane divided by a planar representation of this graph?
i➥ 6
ii ➥ 8
iii ➥ 12
iv ➥ 10

Show Answer With Best Explanation

Answer: II
Explanation: Upload Soon

More DiscussionExplanation On YouTubeLearn Topic WiseHelp-Line

Q.54➡ | NET June 2019
The value of the derivative of the sigmoid function given by
f(x)= 1 / (1+e(-2x)) at x=0 is
i➥ 0
ii ➥ 1/2
iii ➥ 1/4
iv ➥

Show Answer With Best Explanation

Answer: II
Explanation: Upload Soon

More DiscussionExplanation On YouTubeLearn Topic WiseHelp-Line

Q.55➡ | NET June 2019
Consider the poset ({3,5,9,15,24,45},|).
Which of the following is correct for the given poset?
i➥ There exists a greatest element and a least element
ii ➥ There exists a greatest element but not a least element
iii ➥ There exists a least element but not a greatest element
iv ➥ There does not exist a greatest element and a least element

Show Answer With Best Explanation

Answer: IV
Explanation: Upload Soon

More DiscussionExplanation On YouTubeLearn Topic WiseHelp-Line

Q.56➡ | NET June 2019
Consider the following properties with respect to a flow network G=(V,E) in which a flow is a real-valued function f:VxV→ R
P1 : For all u,vεV, f(u,v)= -f(v,u)
P2 : ΣvεV f(u,v)=0 for all uεV

Which one of the following is/are correct?
i➥ Only P1
ii ➥ Only P2
iii ➥ Both P1 and P2
iv ➥ Neither P1 nor P2

Show Answer With Best Explanation

Answer: I
Explanation: Upload Soon

More DiscussionExplanation On YouTubeLearn Topic WiseHelp-Line

Q.57➡ | NET June 2019
A web application and its support environment has not been fully fortified against attack. Web engineers estimate that the likelihood of repelihood an attack is only 30 percent. The application does not contain sensitive or controversial information, so the threat probability is 25 percent. What is the integrity of the web application?
i➥ 0.625
ii ➥ 0.725
iii ➥ 0.775
iv ➥ 0.825

Show Answer With Best Explanation

Answer: IV
Explanation: Upload Soon

More DiscussionExplanation On YouTubeLearn Topic WiseHelp-Line

Q.58➡ | NET June 2019
For which values of m and n does the complete bipartite graph km,n have a Hamilton circuit?
i➥ m≠n, m, n≥2
ii ➥ m≠n, m, n≥3
iii ➥ m=n, m, n≥2
iv ➥ m=2, m, n≥3

Show Answer With Best Explanation

Answer: III
Explanation: Upload Soon

More DiscussionExplanation On YouTubeLearn Topic WiseHelp-Line

Q.59➡ | NET June 2019
How many cards must be selected from a standard deck of 52 cards to guarantee that at least three hearts are present among them?
i➥ 9
ii ➥ 13
iii ➥ 17
iv ➥ 42

Show Answer With Best Explanation

Answer: IV
Explanation: Upload Soon

More DiscussionExplanation On YouTubeLearn Topic WiseHelp-Line

Q.60➡ | NET June 2019
Find the zero-one matrix of the transitive closure of the relation given by matrix A:
Find the zero-one matrix ofthe transitive closure of the relation given by matrix A: A)	1 B)	2 C)	3 D)	4

1) Find the zero-one matrix ofthe transitive closure of the relation given by matrix A: A)	1 B)	2 C)	3 D)	4

2) Find the zero-one matrix ofthe transitive closure of the relation given by matrix A: A)	1 B)	2 C)	3 D)	4

3) Find the zero-one matrix ofthe transitive closure of the relation given by matrix A: A)	1 B)	2 C)	3 D)	4

4) Find the zero-one matrix ofthe transitive closure of the relation given by matrix A: A)	1 B)	2 C)	3 D)	4
i➥ 1
ii ➥ 2
iii ➥ 3
iv ➥ 4

Show Answer With Best Explanation

Answer: I
Explanation: Upload Soon

More DiscussionExplanation On YouTubeLearn Topic WiseHelp-Line

Q.61➡ | NET June 2019
Which of the following is principal conjunctive normal form for [(pVq) ∧ ~p → ~q] ?
i ➥ pV~q
ii ➥ pVq
iii ➥ ~p Vq
iv ➥ ~p V ~q

Show Answer With Best Explanation

Answer: I

Explanation: Upload soon

More DiscussionExplanation On YouTube Learn Topic Wise Help-Line

Q.62➡ | NET June 2019
Match List-I with List-II
Match List-I with List-II List-I                          List-II (a) p → q                (i) ¬(q → ¬p) (b) p v q                  (ii) p ∧ ¬q (c) p ∧ q                  (iii) ¬p → q (d) ¬(p → q)            (iv) ¬p v q Choose the correct option from those given below: A)	(a)-(ii);(b)-(iii);(c)-(i);(d)-(iv) B)	(a)-(ii);(b)-(i);(c)-(iii);(d)-(iv) C)	(a)-(iv);(b)-(i);(c)-(iii);(d)-(ii) D)	(a)-(iv);(b)-(iii);(c)-(i);(d)-(ii)
Choose the correct option from those given below:
i ➥ (a)-(ii);(b)-(iii);(c)-(i);(d)-(iv)
ii ➥ (a)-(ii);(b)-(i);(c)-(iii);(d)-(iv)
iii ➥ (a)-(iv);(b)-(i);(c)-(iii);(d)-(ii)
iv ➥ (a)-(iv);(b)-(iii);(c)-(i);(d)-(ii)

Show Answer With Best Explanation

Answer: IV
Explanation: Upload soon

More DiscussionExplanation On YouTubeBoolean Algebra Help-Line

Q.63➡ | NET December 2018
Consider the vocabulary with only four propositions A,B,C and D. How many models are there for the following sentence?
( ⌐ A ∨ ⌐ B ∨ ⌐ C ∨ ⌐ D)
i➥ 8
ii ➥ 7
iii ➥ 15
iv ➥ 16

Show Answer With Best Explanation

Answer: III
Explanation: Upload Soon

More DiscussionExplanation On YouTubeLearn Topic WiseHelp-Line

Q.64➡ | NET December 2018
The number of substrings that can be formed from string given by “a d e f b g h n m p” is
i➥ 10
ii ➥ 45
iii ➥ 56
iv ➥ 55

Show Answer With Best Explanation

Answer: III
Explanation: Upload Soon

More DiscussionExplanation On YouTubeLearn Topic WiseHelp-Line

Q.65➡ | NET December 2018
Consider the following statements related to AND-OR Search algorithm.
S 1 : A solution is a subtree that has a goal node at every leaf.
S 2 : OR nodes are analogous to the branching in a deterministic environment
S 3 : AND nodes are analogous to the branching in a non-deterministic environment.

Which one of the following is true referencing the above statements?
Choose the correct answer from the code given below:
i➥ S1- False, S2- True, S3- True
ii ➥ S1- True, S2- True, S3- True
iii ➥ S1- False, S2- True, S3- False
iv ➥ S1- True, S2- True, S3- False

Show Answer With Best Explanation

Answer: II
Explanation: Upload Soon

More DiscussionExplanation On YouTubeLearn Topic WiseHelp-Line

Q.66➡ | NET December 2018
The K-coloring of an undirected graph G=(V,E) is a function C: V➝{0,1,……,K-1} such that c(u)≠c(v) for every edge (u,v) ∈ E Which of the following is not correct?
i➥ G has no cycles of odd length
ii ➥ G has cycle of odd length
iii ➥ G is 2-colorable
iv ➥ G is bipartite

Show Answer With Best Explanation

Answer: II
Explanation: Upload Soon

More DiscussionExplanation On YouTubeLearn Topic WiseHelp-Line

Q.67➡ | NET December 2018
If a graph (G) has no loops or parallel edges and if the number of vertices(n) in the graph is n≥3, then the graph G is Hamiltonian if
(i) deg(v) ≥n/3 for each vertex v
(ii) deg(v) + deg(w) ≥ n whenever v and w are not connected by an edge.
(iii) E (G) ≥ 1/3 (n − 1 )(n − 2 ) + 2
i➥ (i) and (iii) only
ii ➥ (ii) and (iii) only
iii ➥ (iii) only
iv ➥ (ii) only

Show Answer With Best Explanation

Answer: IV
Explanation: Upload Soon

More DiscussionExplanation On YouTubeLearn Topic WiseHelp-Line

Q.68➡ | NET December 2018
A survey has been conducted on methods of commuter travel. Each respondent was asked to check Bus, Train or Automobile as a major methods of travelling to work. More than one answer was permitted. The results reported were as follows : Bus 30 people; Train 35 people; Automobile 100 people; Bus and Train 15 people; Bus and Automobile 15 people; Train and Automobile 20 people; and all the three methods 5 people. How many people complete the survey form ?
i➥ 160
ii ➥ 120
iii ➥ 165
iv ➥ 115

Show Answer With Best Explanation

Answer: II
Explanation: Upload Soon

More DiscussionExplanation On YouTubeLearn Topic WiseHelp-Line

Q.69➡ | NET December 2018
A full joint distribution for the Toothache, Cavity and Catch is given in the table below:
A full joint distribution for the Toothache, Cavity and Catch is given in the table below:  Which is the probability of Cavity, given evidence of Toothache ? A)	< 0.2, 0.8 > B)	< 0.6, 0.8 > C)	< 0.4, 0.8 > D)	< 0.6, 0.4 >
Which is the probability of Cavity, given evidence of Toothache ?
i➥ < 0.2, 0.8 >
ii ➥ < 0.6, 0.8 >
iii ➥ < 0.4, 0.8 >
iv ➥ < 0.6, 0.4 >

Show Answer With Best Explanation

Answer: IV
Explanation: Upload Soon

More DiscussionExplanation On YouTubeLearn Topic WiseHelp-Line

Q.70➡ | NET December 2018
In mathematical logic, which of the following are statements ?
(i) There will be snow in January
(ii) What is the time now ?
(iii) Today is Sunday
(iv) You must study Discrete Mathematics.
i➥ (iii) and (iv)
ii ➥ (i) and (ii)
iii ➥ (i) and (iii)
iv ➥ (ii) and (iv)

Show Answer With Best Explanation

Answer: III
Explanation: Upload Soon

More DiscussionExplanation On YouTubeLearn Topic WiseHelp-Line

Q.71➡ | NET December 2018
Consider the statements below :
“ There is a country that borders both India and Nepal. “
Which of the following represents the above sentence correctly ?
i➥ ∃c Border(Country(c), India ∧ Nepal)
ii ➥ ∃c Country(c) ∧ Border(c, India) ∧ Border(c, Nepal)
iii ➥ [∃c Country(c)] ⇒ [Border(c,India) ∧ Border(c, Nepal)]
iv ➥ ∃c Country(c) ⇒ [ Border(c, India) ∧ Border(c, Nepal)]

Show Answer With Best Explanation

Answer: II
Explanation: Upload Soon

More DiscussionExplanation On YouTubeLearn Topic WiseHelp-Line

Q.72➡ | NET December 2018
A box contains six red balls and four green balls. Four balls are selected at random from the box. What is the probability that two of the selected balls will be red and two will be green?
i➥ 1/35
ii ➥ 1/14
iii ➥ 1/9
iv ➥ 3/7

Show Answer With Best Explanation

Answer: IV
Explanation: Upload Soon

More DiscussionExplanation On YouTubeLearn Topic WiseHelp-Line

Q.73➡ | NET June 2018
In a multi-user operating system, 30 requests are made to use a particular resource per hour, on an average. The probability that no requests are made in 40 minutes, when arrival pattern is a poisson distribution, is
i➥ e -15
ii ➥ 1-e -15
iii ➥ 1-e -20
iv ➥ e -20

Show Answer With Best Explanation

Answer: IV
Explanation: Upload Soon

More DiscussionExplanation On YouTubeLearn Topic WiseHelp-Line

Q.74➡ | NET June 2018
Consider the following English sentence : “Agra and Gwalior are both in India”. A student has written a logical sentence for the above English sentence in First-Order Logic using predicate In(x, y), which means x is in y, as follows : In(Agra, India) ⋁ In(Gwalior, India) Which one of the following is correct with respect to the above logical sentence ?
i➥ It is syntactically valid but does not express the meaning of the English sentence.
ii ➥ It is syntactically valid and expresses the meaning of the English sentence also.
iii ➥ It is syntactically invalid but expresses the meaning of the English sentence.
iv ➥ It is syntactically invalid and does not express the meaning of the English sentence.

Show Answer With Best Explanation

Answer: I
Explanation: Upload Soon

More DiscussionExplanation On YouTubeLearn Topic WiseHelp-Line

Q.75➡ | NET June 2018
Consider the set of all possible five-card poker hands dealt fairly from a standard deck of fifty-two cards. How many atomic events are there in the joint probability distribution?
i➥ 2,598,960
ii ➥ 3,468,960
iii ➥ 3,958,590
iv ➥ 2,645,590

Show Answer With Best Explanation

Answer: I
Explanation: Upload Soon

More DiscussionExplanation On YouTubeLearn Topic WiseHelp-Line

Q.76➡ | NET June 2018
E is the number of edges in the graph and f is maximum flow in the graph. When the capacities are integers, the runtime of Ford-Fulkerson algorithm is bounded by:
i➥ O(E∗f)
ii ➥ O(E 2 ∗f)
iii ➥ O(E∗f 2 )
iv ➥ None of the above

Show Answer With Best Explanation

Answer: I
Explanation: Upload Soon

More DiscussionExplanation On YouTubeLearn Topic WiseHelp-Line

Q.77➡ | NET June 2018
Digital data received from a sensor can fill up 0 to 32 buffers. Let the sample space be S={0, 1, 2, ………., 32} where the sample j denote that j of the buffers are full and P (i) = (1/561)(33 − i ) . Let A denote the event that the even number of buffers are full. Then p(A) is:
i➥ 0.515
ii ➥ 0.785
iii ➥ 0.758
iv ➥ 0.485

Show Answer With Best Explanation

Answer: I
Explanation: Upload Soon

More DiscussionExplanation On YouTubeLearn Topic WiseHelp-Line

Q.78➡ | NET June 2018
The equivalence of ¬∃ x Q(x) is:
i➥ ∃x ¬Q(x)
ii ➥ ∀x ¬Q(x)
iii ➥ ¬∃ x ¬Q(x)
iv ➥ ∀x Q(x)

Show Answer With Best Explanation

Answer: II
Explanation: Upload Soon

More DiscussionExplanation On YouTubeLearn Topic WiseHelp-Line

Q.79➡ | NET June 2018
If A i = {−i, … −2,−1, 0, 1, 2, . . . . . i} then If A i = {−i, ... −2,−1, 0, 1, 2, . . . . . i} then is:  A	Z B	Q C	R D	C is:
i➥ Z
ii ➥ Q
iii ➥ R
iv ➥ C

Show Answer With Best Explanation

Answer: I
Explanation: Upload Soon

More DiscussionExplanation On YouTubeLearn Topic WiseHelp-Line

Q.80➡ | NET June 2018
If A i = {−i, … −2,−1, 0, 1, 2, . . . . . i} then If A i = {−i, ... −2,−1, 0, 1, 2, . . . . . i} then is:  A	Z B	Q C	R D	C is:
i➥ Z
ii ➥ Q
iii ➥ R
iv ➥ C

Show Answer With Best Explanation

Answer: I
Explanation: Upload Soon

More DiscussionExplanation On YouTubeLearn Topic WiseHelp-Line

Q.81➡ | NET June 2018
Which of the relations on {0, 1, 2, 3} is an equivalence relation?
i➥ { (0, 0) (0, 2) (2, 0) (2, 2) (2, 3) (3, 2) (3, 3) }
ii ➥ { (0, 0) (1, 1) (2, 2) (3, 3) }
iii ➥ { (0, 0) (0, 1) (0, 2) (1, 0) (1, 1) (1, 2) (2, 0) }
iv ➥ { (0, 0) (0, 2) (2, 3) (1, 1) (2, 2) }

Show Answer With Best Explanation

Answer: II
Explanation: Upload Soon

More DiscussionExplanation On YouTubeLearn Topic WiseHelp-Line

Q.82➡ | NET June 2018
Which of the following is an equivalence relation on the set of all functions from Z to Z?
i➥ { (f , g ) | f (x) − g (x) = 1 ∀ x ∈ Z }
ii ➥ { (f , g ) | f (0) = g (0) or f (1) = g (1)}
iii ➥ { (f , g ) | f (0) = g (1) and f (1) = g (0)}
iv ➥ { (f , g ) | f (x) − g (x) = k f or some k ∈ Z }

Show Answer With Best Explanation

Answer: IV
Explanation: Upload Soon

More DiscussionExplanation On YouTubeLearn Topic WiseHelp-Line

Q.83➡ | NET June 2018
Which of the following statements is true ?
i➥ (Z, ≤ ) is not totally ordered
ii ➥ The set inclusion relation ⊆ is a partial ordering on the power set of a set S
iii ➥ (Z, ≠ ) is a poset
iv ➥ The directed graph Which of the following statements is true ? A	(Z, ≤ ) is not totally ordered B	The set inclusion relation ⊆ is a partial ordering on the power set of a  set S C	(Z, ≠ ) is a poset D	The directed graph   is not a partial orderis not a partial order

Show Answer With Best Explanation

Answer: II
Explanation: Upload Soon

More DiscussionExplanation On YouTubeLearn Topic WiseHelp-Line

Q.84➡ | NET November 2017 Paper 2
If the time is now 4 O’clock, what will be the time after 101 hours from now?
i➥ 9 O’Clock
ii ➥ 8 O’Clock
iii ➥ 5 O’Clock
iv ➥ 4 O’Clock

Show Answer With Best Explanation

Answer: I
Explanation: Upload Soon

More DiscussionExplanation On YouTubeLearn Topic WiseHelp-Line

Q.85➡ | NET November 2017 Paper 2
Let A = Let A =   and B =   Find the boolean product A⊙B of the two matrices. and B =Let A =   and B =   Find the boolean product A⊙B of the two matrices.
Find the boolean product A⊙B of the two matrices.
i➥ Let A =   and B =   Find the boolean product A⊙B of the two matrices.
ii ➥ Let A =   and B =   Find the boolean product A⊙B of the two matrices.
iii ➥ Let A =   and B =   Find the boolean product A⊙B of the two matrices.
iv ➥ Let A =   and B =   Find the boolean product A⊙B of the two matrices.

Show Answer With Best Explanation

Answer: I
Explanation: Upload Soon

More DiscussionExplanation On YouTubeLearn Topic WiseHelp-Line

Q.86➡ | NET November 2017 Paper 2
How many distinguishable permutations of the letters in the word BANANA are there?
i➥ 720
ii ➥ 120
iii ➥ 60
iv ➥ 360

Show Answer With Best Explanation

Answer: III
Explanation: Upload Soon

More DiscussionExplanation On YouTubeLearn Topic WiseHelp-Line

Q.87➡ | NET November 2017 Paper 2
Let P and Q be two propositions, ¬ (P ↔ Q) is equivalent to:
i➥ P ↔ ¬ Q
ii ➥ ¬ P ↔ Q
iii ➥ ¬ P ↔ ¬ Q
iv ➥ Q → P

Show Answer With Best Explanation

Answer: I,II Both (Marks to all)
Explanation: Upload Soon

More DiscussionExplanation On YouTubeLearn Topic WiseHelp-Line

Q.88➡ | NET November 2017 Paper 2
Negation of the proposition ∃x H(x) is:
i➥ ∃x ⌐H(x)
ii ➥ ∀x ⌐H(x)
iii ➥ ∀x H(x)
iv ➥ ⌐x H(x)

Show Answer With Best Explanation

Answer: II
Explanation: Upload Soon

More DiscussionExplanation On YouTubeLearn Topic WiseHelp-Line

Q.89➡ | NET November 2017 Paper 3
Let P, Q, R and S be Propositions. Assume that the equivalences P ⇔ (Q ∨ ¬ Q) and Q ⇔ R hold. Then the truth value of the formula (P ∧ Q) ⇒ ((P ∧ R) ∨ S) is always:
i➥ True
ii ➥ False
iii ➥ Same as truth table of Q
iv ➥ Same as truth table of S

Show Answer With Best Explanation

Answer: I
Explanation: Upload Soon

More DiscussionExplanation On YouTubeLearn Topic WiseHelp-Line

Q.90➡ | NET November 2017 Paper 3
“If X, then Y unless Z” is represented by which of the following formulae in propositional logic?
i➥ (X ∧ Y) → ¬ Z
ii ➥ (X ∧ ¬ Z) → Y
iii ➥ X → (Y ∧ ¬ Z)
iv ➥ Y → (X ∧ ¬ Z)

Show Answer With Best Explanation

Answer: II
Explanation: Upload Soon

More DiscussionExplanation On YouTubeLearn Topic WiseHelp-Line

error: Content is protected !!
Open chat
1
Hi,how Can We Help You ?